pt 34 s3 #5 Forum

Prepare for the LSAT or discuss it with others in this forum.
Post Reply
HOV

New
Posts: 61
Joined: Mon May 31, 2010 3:31 pm

pt 34 s3 #5

Post by HOV » Sat Aug 07, 2010 9:41 pm

calling all who have access to this test

alright so i think any confusion i'm experiencing with this question stems from my understanding of necessary assumption vs. sufficient assumption.

could somebody please take a minute to explain why the answer is not A? is it because it is not NECESSARY to ensure the arguments validity? i feel like if either A or D is assumed, the columnists argument is well served...what's the difference between the two that i'm missing?

thanks in advance.

EDIT:SHOOT, i'm sorry this was in reference to section 3 not section 2. my mistake.
Last edited by HOV on Sat Aug 07, 2010 10:16 pm, edited 2 times in total.

User avatar
Hannibal

Gold
Posts: 2211
Joined: Mon Jul 12, 2010 12:00 pm

Re: pt 34 s2 #5

Post by Hannibal » Sat Aug 07, 2010 10:13 pm

The contention in the question is over why math is different from science, reading and writing.

A isn't useful since it puts math and reading in the same category, even though they had opposite results.
D doesn't really say anything since there isn't any connection in the question between usefulness, interest, video graphics, etc and math.

B is correct because it singles out math as especially useful with a property that could also be applied to a lesser extent to science.

User avatar
Hannibal

Gold
Posts: 2211
Joined: Mon Jul 12, 2010 12:00 pm

Re: pt 34 s3 #5

Post by Hannibal » Sat Aug 07, 2010 10:20 pm

I got this one right, but I can see why A is appealing.

I eliminated it because it jumps reasoning a little bit. A is essentially restating the conclusion, but with too strong of language. "Anyone" is not at all assumed.

HOV

New
Posts: 61
Joined: Mon May 31, 2010 3:31 pm

Re: pt 34 s3 #5

Post by HOV » Sat Aug 07, 2010 10:37 pm

yeah now that i think about it, in this question it seems essential to note that the columnist refers to widespread reliance whereas answer choice A seems like it too abruptly narrows things down to any given individual.

thanks H.

ly2010

Bronze
Posts: 103
Joined: Sat Aug 07, 2010 8:30 pm

Re: pt 34 s3 #5

Post by ly2010 » Sat Aug 07, 2010 10:44 pm

I got this one wrong the first time I did it as well. The missing link is between the necessary bond and participating in groups outside the family.

Want to continue reading?

Register now to search topics and post comments!

Absolutely FREE!


User avatar
yzero1

Bronze
Posts: 185
Joined: Thu May 27, 2010 4:33 pm

Re: pt 34 s3 #5

Post by yzero1 » Sat Aug 07, 2010 10:47 pm

From the stimulus:

Democratic society -> strong bonds of mutual trust -> participation in civic organizations, political arties, + other groups outside family

Conclusion: Reliance on movies and media for entertainment -> corrosive effect on democracy

Basically, we need something that, if negated, will destroy the conclusion.

A) basically forms this diagram: rely on movies + media for entertainment -> - strong bonds of mutual trust. This does sufficiently justify the conclusion because rely on movies -> -strong bonds of trust -> - democratic society via the contrapositive. However, you must remember that the stimulus is asking for a NECESSARY assumption, and not merely something that justifies the conclusion if assumed. If you negate this answer choice, you get:

"Not everyone who relies on movies and electronic media for entertainment is unable to form a strong bond of mutual trust with a citizen".

This does not completely destroy the conclusion because the widespread reliance on movies and electronic media can STILL HAVE a corrosive effect on democracy as long as some/many/most of the people relying on the aforemetioned cannot form strong bonds of mutual trust.

Now try negating (D). "Relying on movies + elec. media for entertainment DOES NOT generally make people less likely to participate in groups outside their families". If this is true, then relying on moves + electronic media does not go against one of the necessary conditions for democracy and the author's argument fails. Thus, this is the correct answer.

HOV

New
Posts: 61
Joined: Mon May 31, 2010 3:31 pm

Re: pt 34 s3 #5

Post by HOV » Sun Aug 08, 2010 11:17 am

thanks guys, i got it.

Want to continue reading?

Register for access!

Did I mention it was FREE ?


Post Reply

Return to “LSAT Prep and Discussion Forum”